subject
Physics, 12.08.2020 17:01 stormhorn5108

A circular loop in the plane of a paper lies in a 0.45 T magnetic field pointing into the paper. The loop's diameter changes from 17.0 cm to 6.0 cm in 0.53 s. A) Determine the direction of the induced current.
B) Determine the magnitude of the average induced emf.
C) If the coil resistance is 2.5 Ω, what is the average induced current?

ansver
Answers: 2

Another question on Physics

question
Physics, 22.06.2019 11:20
Wave functions describe orbitals in a hydrogen atom. each function is characterized by 3 quantum numbers: n, l, and ml. if the value of n = 2: the quantum number l can have values from to . the total number of orbitals possible at the n = 2 energy level is .
Answers: 3
question
Physics, 22.06.2019 19:40
Two charged particles, q1 and q2, are located on the x-axis, with q1 at the origin and q2 initially at x1 = 12.2 mm. in this configuration, q1 exerts a repulsive force of 2.62 µn on q2. particle q2 is then moved to x2 = 18.0 mm. what is the force (magnitude and direction) that q2 exerts on q1 at this new location? (give the magnitude in µn.)
Answers: 1
question
Physics, 22.06.2019 19:50
Aleaky 10-kg bucket is lifted from the ground to a height of 15 m at a constant speed with a rope that weighs 0.7 kg/m. initially the bucket contains 45 kg of water, but the water leaks at a constant rate and finishes draining just as the bucket reaches the 15-m level. find the work done. (use 9.8 m/s2 for g.) show how to approximate the required work by a riemann sum. (let x be the height in meters above the ground. enter xi* as xi.) lim n → ∞ n (9.8)(45−3 xi) i = 1 δx express the work as an integral. 0 dx evaluate the integral. (round your answer to the nearest integer.) j
Answers: 3
question
Physics, 22.06.2019 20:30
Your friend increases her force on the desk by 30n . she doesn't change the direction of her push . what happens to the net force on the desk .will the desk accelerate
Answers: 2
You know the right answer?
A circular loop in the plane of a paper lies in a 0.45 T magnetic field pointing into the paper. The...
Questions
question
History, 11.07.2019 20:20
Questions on the website: 13722367